3
$\begingroup$

Suppose we have a finite group $G$ with subgroup $H$, a representation $\rho_V$ of $H$ on a finite-dimensional vector space $V$, and an $H$-invariant inner product on $V$:

$$\forall x,y\in V, h\in H,\enspace \langle\rho_V(h)x, \rho_V(h)y\rangle = \langle x,y\rangle$$

We will write $V_I$ for the direct sum of $\lvert G:H \rvert$ copies of $V$: $$V_I = \oplus_{i=1}^{\lvert G:H \rvert} V$$ We define a map $L_i$ that lifts $V$ into the $i$th copy in the direct sum: $$L_i: V\to V_I\\ L_i v = 0 \oplus 0 \oplus ... \underbrace{v}_{i\text{th summand}} \oplus 0 + ...$$ We extend the inner product on $V$ to one on $V_I$:

$$ \langle \sum_i L_i x_i, \sum_j L_j y_j \rangle = \sum_i \langle x_i, y_i \rangle$$ From each left coset $K_i$ of $H$ in $G$ we pick an element $k_i$, so $K_i=k_i H$. We then have an induced representation $\rho_I$ of the group $G$ on $V_I$: $$\rho_I(g) \sum_i L_i v_i = \sum_i L_{j(g,i)} \rho_V(k_{j(g,i)}^{-1} g k_i) v_i$$

where $j(g,i)$ is the index of the coset $K_i$ to which $g k_i$ belongs.

Now, suppose we have an irreducible representation $\rho_W$ of $G$ on some finite vector space $W$. The Frobenius reciprocity theorem says that $Hom_H(W,V)$, the space of $H$-intertwiners from $W$ to $V$, i.e. the space of maps $S$ that satisfy:

$$S: W\to V\\ \forall h\in H,\enspace S \rho_W(h) = \rho_V(h) S$$ is isomorphic to the space $Hom_G(W,V_I)$ of $G$-intertwiners from $W$ to $V_I$, i.e. the space of maps $T$ that satisfy:

$$T: W\to V_I\\ \forall g\in G,\enspace T \rho_W(g) = \rho_I(g) T$$

Indeed, given an intertwiner $S\in Hom_H(W,V)$ we can easily construct an intertwiner $T_S\in Hom_G(W,V_I)$:

$$T_S w = \sum_i L_i S \rho_W(k_i^{-1}) w$$

As we vary $S$ over any basis of $Hom_H(W,V)$, the associated map $T_S$ will map $W$ into distinct subspaces of $V_I$, each of which is invariant under the action of $\rho_I$, and each of which has the property that the restriction of $\rho_I$ to that subspace is equivalent to $\rho_W$.

My question is: supposing the dimension of $Hom_H(W,V)$ is greater than 1, does there exist an "easy" strategy to choose a basis $\{S_1,S_2,...\}$ of $Hom_H(W,V)$ such that the subspaces $T_{S_1}(W), T_{S_2}(W), ...$ of $V_I$ will be mutually orthogonal?

By "easy", I mean something less computationally demanding than simply finding all the subspaces by means of an arbitrary basis for $Hom_H(W,V)$, and then decomposing their direct sum into orthogonal invariant subspaces. In other words, I am seeking to leverage the fact that $Hom_H(W,V)$ is a lower-dimensional space than the direct sum of the $T_{S_i}(W)$ in order to carry out a less demanding procedure to achieve the same result.

Edited to add: One easy way to get a basis of $Hom_H(W,V)$ is to take a sufficient number of linearly independent maps $S^{(0)}_i: W\to V$ and average over the subgroup $H$:

$$S_i = \frac{1}{|H|}\sum_{h\in H} \rho_V(h) S^{(0)}_i \rho_W(h^{-1})$$

So by starting with a basis of (non-intertwining) maps from $W$ to $V$, you can project as many as required into $Hom_H(W,V)$ to obtain a basis.

My (possibly naive) hope is that there might be some way of modifying this construction to lead directly to a basis with the property I'm seeking.

$\endgroup$
6
  • 1
    $\begingroup$ If you want to solve it on a computer, then it is easy to reformulate the problem to equations for $S$. $$ \forall a,b \; \forall u,v\in W : \sum_i \langle S_a\rho(k_i^{-1})u, S_b\rho(k_i^{-1})v\rangle = 0 $$ $\endgroup$ Aug 4, 2014 at 13:34
  • $\begingroup$ OK, so that would be a large system of simultaneous quadratics in the components of the $S_a$, with $\frac{d(d-1)n^2}{2}$ equations, where $d$ is the dimension of $Hom_H(W,V)$ and $n$ is the dimension of $W$. I was hoping more for some kind of direct construction. $\endgroup$
    – Greg Egan
    Aug 4, 2014 at 14:00
  • $\begingroup$ I was thinking you can build $S_a$ iteratively. If you have a linearly independent set $\{S_1, \ldots, S_a\}$ you can try to solve the system of linear equations: $S_{a+1}$ is an intertwiner that is "orthogonal" to $\{S_1, \ldots, S_a\}$ such that $\{S_1, \ldots, S_{a+1}\}$ is linearly independent. $\endgroup$ Aug 4, 2014 at 15:48
  • 1
    $\begingroup$ Actually, you can see from that equation that if $S_1$ and $S_2$ have orthogonal images, then the same is true for $T_{S_1}$ and $T_{S_2}$. You already have the projection $\mathrm{Hom}(W,V) \to \mathrm{Hom}_H(W,V)$ which should give you all "orthogonal" intertwiners as soon as you introduce appropriate scalar product on $\mathrm{Hom}(W,V)$ that will turn "orthogonal" (as in orthogonal image) to really orthogonal. I guess $\langle A,B \rangle = \mathrm{Tr} A\overline{B}^T$ will do. $\endgroup$ Aug 4, 2014 at 17:09
  • 1
    $\begingroup$ The catch is that the projection $\mathrm{Hom}(W,V)\to \mathrm{Hom}_H(W,V)$ doesn't preserve orthogonality, so you can't start with an orthogonal basis of $\mathrm{Hom}(W,V)$ and then project it. So I suppose it's necessary to project to $\mathrm{Hom}_H(W,V)$ to get a non-orthogonal basis, and then use Gram-Schmidt or Householder to make it orthogonal. $\endgroup$
    – Greg Egan
    Aug 5, 2014 at 0:26

1 Answer 1

1
$\begingroup$

I'm going to assume that the field is $\mathbb{C}$ (what I'm about to say will be wrong over $\mathbb{R}$. The space $Hom_H(W,V)$ has an inner product given by $\langle f,g\rangle=\sum \langle f(e_i),g(e_i)\rangle$ for $e_i$ an orthonormal basis of $W$. If you write $f,g$ as matrices in orthonormal bases of $V$ and $W$, this will be the component-wise dot product of matrices; taking adjoint will identify $ Hom_H(W,V)\cong Hom_H(V,W)$ (this map will only be semi-linear) and the inner product is just the trace of the composition $f^\dagger g\colon W\to W$. You can easily check that taking induction will just multiply this by the index $[G:H]$.

By Schur's lemma, $f^\dagger g$ is a scalar matrix, so $\langle f,g\rangle=0$ if and only if $f^\dagger g=0$; this will precisely happen if $f$ and $g$ have orthogonal images (by the definition of adjoint). So, you just want an orthonormal basis of $Hom_H(W,V)$ or $Hom_G(W,V_I)$, you just do Gram-Schmidt.

$\endgroup$
0

Your Answer

By clicking “Post Your Answer”, you agree to our terms of service and acknowledge you have read our privacy policy.

Not the answer you're looking for? Browse other questions tagged or ask your own question.